[skip-to-content]
\(\newcommand{\identity}{\mathrm{id}} \newcommand{\notdivide}{{\not{\mid}}} \newcommand{\notsubset}{\not\subset} \newcommand{\lcm}{\operatorname{lcm}} \newcommand{\gf}{\operatorname{GF}} \newcommand{\inn}{\operatorname{Inn}} \newcommand{\aut}{\operatorname{Aut}} \newcommand{\Hom}{\operatorname{Hom}} \newcommand{\cis}{\operatorname{cis}} \newcommand{\chr}{\operatorname{char}} \newcommand{\Null}{\operatorname{Null}} \renewcommand{\gcd}{\operatorname{mcd}} \renewcommand{\lcm}{\operatorname{mcm}} \renewcommand{\deg}{\operatorname{gr}} \newcommand{\lt}{<} \newcommand{\gt}{>} \newcommand{\amp}{&} \)

Sección2.3Ejercicios

1

Demuestre que

\begin{equation*} 1^2 + 2^2 + \cdots + n^2 = \frac{n(n + 1)(2n + 1)}{6} \end{equation*}

para \(n \in {\mathbb N}\text{.}\)

2

Demuestre que

\begin{equation*} 1^3 + 2^3 + \cdots + n^3 = \frac{n^2(n + 1)^2}{4} \end{equation*}

para \(n \in {\mathbb N}\text{.}\)

3

Demuestre que \(n! \gt 2^n\) para \(n \geq 4\text{.}\)

4

Demuestre que

\begin{equation*} x + 4x + 7x + \cdots + (3n - 2)x = \frac{n(3n - 1)x}{2} \end{equation*}

para todo \(n \in {\mathbb N}\text{.}\)

5

Demuestre que \(10^{n + 1} + 10^n + 1\) es divisible por 3 para todo \(n \in {\mathbb N}\text{.}\)

6

Demuestre que \(4 \cdot 10^{2n} + 9 \cdot 10^{2n - 1} + 5\) es divisible por 99 para todo \(n \in {\mathbb N}\text{.}\)

7

Muestre que

\begin{equation*} \sqrt[n]{a_1 a_2 \cdots a_n} \leq \frac{1}{n} \sum_{k = 1}^{n} a_k. \end{equation*}
8

Demuestre la regla de Leibniz para \(f^{(n)} (x)\text{,}\) donde \(f^{(n)}\) es la \(n\)-ésima derivada de \(f\text{;}\) es decir, muestre que

\begin{equation*} (fg)^{(n)}(x) = \sum_{k = 0}^{n} \binom{n}{k} f^{(k)}(x) g^{(n - k)}(x). \end{equation*}
9

Use inducción para demostrar que \(1 + 2 + 2^2 + \cdots + 2^n = 2^{n + 1} - 1\) para todo \(n \in {\mathbb N}\text{.}\)

10

Demuestre que

\begin{equation*} \frac{1}{2}+ \frac{1}{6} + \cdots + \frac{1}{n(n + 1)} = \frac{n}{n + 1} \end{equation*}

para todo \(n \in {\mathbb N}\text{.}\)

11

Si \(x\) es un número real no negativo, demuestre que \((1 + x)^n - 1 \geq nx\) para \(n = 0, 1, 2, \ldots\text{.}\)

12Conjunto Potencia

Sea \(X\) un conjunto. Defina el conjunto potencia de \(X\text{,}\) denotado \({\mathcal P}(X)\text{,}\) como el conjunto de todos los subconjuntos de \(X\text{.}\) Por ejemplo,

\begin{equation*} {\mathcal P}( \{a, b\} ) = \{ \emptyset, \{a\}, \{b\}, \{a, b\} \}. \end{equation*}

Para todo entero positivo \(n\text{,}\) muestre que un conjunto con exactamente \(n\) elementos tiene un conjunto potencia con exactamente \(2^n\) elementos.

13

Demuestre que que los dos Principios de Inducción enunciados en la Sección2.1 son equivalentes.

14

Muestre que el Principio del Buen-Orden para los números naturales implica que 1 es el menor número natural. Use este resultado para mostrar que el Principio del Buen-Orden implica el Principio de Inducción; es decir, muestre que si \(S \subset {\mathbb N}\) tal que \(1 \in S\) y \(n + 1 \in S\) cada vez que \(n \in S\text{,}\) entonces \(S = {\mathbb N}\text{.}\)

15

Para cada uno de los siguientes pares de números \(a\) y \(b\text{,}\) calcule \(\gcd(a,b)\) y encuentre enteros \(r\) y \(s\) tales que \(\gcd(a,b) = ra + sb\text{.}\)

  1. 14 y 39

  2. 234 y 165

  3. 1739 y 9923

  4. 471 y 562

  5. 23771 y 19945

  6. \(-4357\) y 3754

16

Sean \(a\) y \(b\) enteros distintos de cero. Si existen enteros \(r\) y \(s\) tales que \(ar + bs =1\text{,}\) muestre que \(a\) y \(b\) son relativamente primos.

17Números de Fibonacci

Los Números de Fibonacci son

\begin{equation*} 1, 1, 2, 3, 5, 8, 13, 21, \ldots. \end{equation*}

Podemos definirlos recursivamente como \(f_1 = 1\text{,}\) \(f_2 = 1\text{,}\) y \(f_{n + 2} = f_{n + 1} + f_n\) para \(n \in {\mathbb N}\text{.}\)

  1. Demuestre que \(f_n \lt 2^n\text{.}\)

  2. Demuestre que \(f_{n + 1} f_{n - 1} = f^2_n + (-1)^n\text{,}\) \(n \geq 2\text{.}\)

  3. Demuestre que \(f_n = [(1 + \sqrt{5}\, )^n - (1 - \sqrt{5}\, )^n]/ 2^n \sqrt{5}\text{.}\)

  4. Muestre que \(\lim_{n \rightarrow \infty} f_n / f_{n + 1} = (\sqrt{5} - 1)/2\text{.}\)

  5. Demuestre que \(f_n\) y \(f_{n + 1}\) son relativamente primos.

18

Sean \(a\) y \(b\) enteros tales que \(\gcd(a,b) = 1\text{.}\) Sean \(r\) y \(s\) enteros tales que \(ar + bs =1\text{.}\) Demuestre que

\begin{equation*} \gcd(a,s) = \gcd(r,b) = \gcd(r,s) = 1. \end{equation*}
19

Sean \(x, y \in {\mathbb N}\) relativamente primos. Si \(xy\) es un cuadrado perfecto, demuestre que \(x\) e \(y\) son ambos cuadrados perfectos.

20

Usando el algoritmo de división, muestre que todo cuadrado perfecto es de la forma \(4k\) o \(4k + 1\) para algún entero no negativo \(k\text{.}\)

21

Supongamos que \(a, b, r, s\) son relativamente primos de a pares y que

\begin{align*} a^2 + b^2 & = r^2\\ a^2 - b^2 & = s^2. \end{align*}

Demuestre que \(a\text{,}\) \(r\text{,}\) y \(s\) son impares y que \(b\) es par.

22

Sea \(n \in {\mathbb N}\text{.}\) Use el algoritmo de división para demostrar que todo entero es congruente mód \(n\) a exactamente uno de los enteros \(0, 1, \ldots, n-1\text{.}\) Concluya que si \(r\) es un entero, entonces hay exactamente un \(s\) en \({\mathbb Z}\) tal que \(0 \leq s \lt n\) y \([r] = [s]\text{.}\) Luego, los enteros están efectivamente particionados por la relación de congruencia mód \(n\text{.}\)

23

Defina el mínimo común múltiplo de dos enteros distintos de cero \(a\) y \(b\text{,}\) denotado por \(\lcm(a,b)\text{,}\) como el entero positivo \(m\) tal que tanto \(a\) como \(b\) dividen a \(m\text{,}\) y si \(a\) y \(b\) dividen a otro entero \(n\text{,}\) entonces \(m\) también divide a \(n\text{.}\) Demuestre que existe un único mínimo común múltiplo para cualquiera dos enteros \(a\) y \(b\) distintos de cero.

24

Si \(d= \gcd(a, b)\) y \(m = \lcm(a, b)\text{,}\) demuestre que \(dm = |ab|\text{.}\)

25

Muestre que \(\lcm(a,b) = ab\) si y solo si \(\gcd(a,b) = 1\text{.}\)

26

Demuestre que \(\gcd(a,c) = \gcd(b,c) =1\) si y solo si \(\gcd(ab,c) = 1\) para todos los enteros \(a\text{,}\) \(b\text{,}\) y \(c\text{.}\)

27

Sean \(a, b, c \in {\mathbb Z}\text{.}\) Demuestre que si \(\gcd(a,b) = 1\) y \(a \mid bc\text{,}\) entonces \(a \mid c\text{.}\)

28

Sea \(p \geq 2\text{.}\) Demuestre que si \(2^p - 1\) es primo, entonces \(p\) también es primo.

29

Demuestre que hay infinitos primos de la forma \(6n + 5\text{.}\)

30

Demuestre que hay infinitos primos de la forma \(4n - 1\text{.}\)

31

Usando el hecho que 2 es primo, muestre que no existen enteros \(p\) y \(q\) tales que \(p^2 = 2 q^2\text{.}\) Demuestre que por lo tanto \(\sqrt{2}\) no puede ser un número racional.